Prove that the matrices have the same rank.

  • Thread starter Thread starter Dafe
  • Start date Start date
  • Tags Tags
    Matrices rank
Click For Summary
SUMMARY

The discussion centers on proving that three matrices derived from a base matrix A have the same rank. The matrices in question are: the original matrix A, the concatenated matrix [A A], and the block matrix [[A A], [A A]]. Through elimination, it is established that the rank of the second matrix remains equal to that of A, and similarly, the third matrix does not introduce new independent vectors, confirming that its rank is also equal to that of A. The conclusion is that all three matrices maintain the same rank as matrix A.

PREREQUISITES
  • Understanding of matrix rank and its implications in linear algebra.
  • Familiarity with matrix operations, specifically row reduction and elimination techniques.
  • Knowledge of block matrices and their properties.
  • Ability to interpret and manipulate mathematical notation related to matrices.
NEXT STEPS
  • Study the concept of matrix rank in detail, focusing on definitions and properties.
  • Learn about row reduction techniques and their applications in determining matrix rank.
  • Explore block matrix operations and their implications on rank and linear independence.
  • Investigate the relationship between linear transformations and matrix rank.
USEFUL FOR

Students of linear algebra, mathematicians, and educators seeking to deepen their understanding of matrix rank and its proof techniques.

Dafe
Messages
144
Reaction score
0

Homework Statement


Prove that the three matrices have the same rank.


<br /> <br /> \left[<br /> \begin{array}{c}<br /> A\\<br /> \end{array}<br /> \right]<br /> <br />

<br /> <br /> \left[<br /> \begin{array}{c}<br /> A &amp; A\\<br /> \end{array}<br /> \right]<br /> <br />

<br /> <br /> \left[<br /> \begin{array}{cc}<br /> A &amp; A\\<br /> A &amp; A\\<br /> \end{array}<br /> \right]<br /> <br />

Homework Equations





The Attempt at a Solution



If elimination is done on the second matrix it will become:
<br /> <br /> \left[<br /> \begin{array}{c}<br /> A &amp; 0\\<br /> \end{array}<br /> \right]<br /> <br />

This means that the rank is still the same as A.

Elimination on the third matrix gives:
<br /> <br /> \left[<br /> \begin{array}{cc}<br /> A &amp; A\\<br /> 0 &amp; 0\\<br /> \end{array}<br /> \right]<br /> <br />

Since no new independent vectors are added, it also has rank A.

I do understand that this is so, but could someone please help me explain this mathematically?

Thanks.
 
Physics news on Phys.org
About the only thing different I would say is to suppose that when
<br /> \left[<br /> \begin{array}{c}<br /> A\\<br /> \end{array}<br /> \right]<br />
is reduced you obtain
<br /> \left[<br /> \begin{array}{c}<br /> R\\<br /> \end{array}<br /> \right]<br />


Then express your reduced forms of the other two matrices in terms of R instead of A. Refer to the number of nonzero rows in R, and you are done.
 
Hi Billy Bob, thanks for the reply.
Here's the way I think you would do it: (just showing one matrix)

<br /> <br /> \left[<br /> \begin{array}{c}<br /> A &amp; A\\<br /> \end{array}<br /> \right]<br /> <br />

<br /> <br /> \left[<br /> \begin{array}{c}<br /> R &amp; 0\\<br /> \end{array}<br /> \right]<br /> <br />

# non zero rows = r for all matrices.
 
Question: A clock's minute hand has length 4 and its hour hand has length 3. What is the distance between the tips at the moment when it is increasing most rapidly?(Putnam Exam Question) Answer: Making assumption that both the hands moves at constant angular velocities, the answer is ## \sqrt{7} .## But don't you think this assumption is somewhat doubtful and wrong?

Similar threads

Replies
6
Views
1K
  • · Replies 5 ·
Replies
5
Views
2K
Replies
0
Views
887
  • · Replies 9 ·
Replies
9
Views
2K
  • · Replies 7 ·
Replies
7
Views
4K
  • · Replies 1 ·
Replies
1
Views
2K
  • · Replies 9 ·
Replies
9
Views
2K
  • · Replies 11 ·
Replies
11
Views
2K
Replies
1
Views
2K
  • · Replies 5 ·
Replies
5
Views
3K